Existence of two disjoint long cycles in graphs (Q2581410): Difference between revisions

From MaRDI portal
Added link to MaRDI item.
ReferenceBot (talk | contribs)
Changed an Item
 
(2 intermediate revisions by 2 users not shown)
Property / MaRDI profile type
 
Property / MaRDI profile type: MaRDI publication profile / rank
 
Normal rank
Property / full work available at URL
 
Property / full work available at URL: https://doi.org/10.1016/j.disc.2005.10.017 / rank
 
Normal rank
Property / OpenAlex ID
 
Property / OpenAlex ID: W2091613024 / rank
 
Normal rank
Property / cites work
 
Property / cites work: Q4094891 / rank
 
Normal rank
Property / cites work
 
Property / cites work: Degree conditions for 2-factors / rank
 
Normal rank
Property / cites work
 
Property / cites work: Some Theorems on Abstract Graphs / rank
 
Normal rank
Property / cites work
 
Property / cites work: Cycles and paths through specified vertices in k-connected graphs / rank
 
Normal rank
Property / cites work
 
Property / cites work: On the existence of disjoint cycles in a graph / rank
 
Normal rank
Property / cites work
 
Property / cites work: On the maximum number of independent cycles in a graph / rank
 
Normal rank
Property / cites work
 
Property / cites work: Q4488568 / rank
 
Normal rank

Latest revision as of 14:52, 11 June 2024

scientific article
Language Label Description Also known as
English
Existence of two disjoint long cycles in graphs
scientific article

    Statements

    Existence of two disjoint long cycles in graphs (English)
    0 references
    0 references
    0 references
    0 references
    0 references
    10 January 2006
    0 references
    Following a conjecture by H. Wang in 2000, Egawa, and others, proved a theorem regarding the existence of \(k\) disjoint cycles in a graph. The authors now show that the 2-cycle version of that theorem holds as well: If \(n \geq 6\), \(h \geq 7\) are integers, and \(G\) is a graph of order \(n\) in which the degree sum of any pair of nonadjacent vertices is at least \(h\), then \(G\) contains two disjoint cycles having \(\geq\min\{ h,n \}\) vertices altogether. The proof requires an exhausting analysis of possible disjoint cycles in a counterexample.
    0 references
    0 references
    cycle
    0 references
    subgraph
    0 references
    vertex-disjoint
    0 references
    0 references